The Student Room Group

STEP Prep Thread 2015

Scroll to see replies

Original post by brianeverit
Yes. when you have 1+x^2 to get rid of.
BY the way, what were your two answers?
This was an interesting one.
Two points P and Q lie within, or on the boundary of, a square of side 1 cm, one corner of which is the point O. Show that the length of at least one of the lines OP,PQ and QO must be less than or equal to (62)(\sqrt6-\sqrt2) cm


That is Q1 2001, right? Already done it, I'm afraid. Any others?!
Original post by DomStaff
That is Q1 2001, right? Already done it, I'm afraid. Any others?!


This exact same question has come up twice in STEP I
Original post by newblood
This exact same question has come up twice in STEP I


What was the other year?
Original post by DomStaff
What was the other year?


I think it was actually the year after it was first written. Let me have a look

EDIT: It was 1988 and 2001. Exact same question (wording and all)
(edited 9 years ago)
Original post by newblood
I think it was actually the year after it was first written. Let me have a look

EDIT: It was 1988 and 2001. Exact same question (wording and all)


No way! Does this happen often?
Original post by DomStaff
No way! Does this happen often?


not often, but it has happened occasionally. i think thats the only step i question its happened in though. 2006 STEP III for example had about 3 questions from the older STEP II/III papers
Original post by newblood
not often, but it has happened occasionally. i think thats the only step i question its happened in though. 2006 STEP III for example had about 3 questions from the older STEP II/III papers


Oh, I see. That provides motivation to do as many papers as possible.
Original post by DomStaff
Oh, I see. That provides motivation to do as many papers as possible.


Certainly do lots of papers: but i wouldnt do it in hope of remembering certain solutions as the chances of a repeated question coming up are still very small...i doubt its happened in any paper in the last 5 years or so at least.
Original post by newblood
Certainly do lots of papers: but i wouldnt do it in hope of remembering certain solutions as the chances of a repeated question coming up are still very small...i doubt its happened in any paper in the last 5 years or so at least.


Good point. Although, you'd kick yourself if a question had came up and that happened to be from the paper you hadn't done!
Original post by shamika
It's a point of principle (and law) - he wrote the book, he's the only one who should be able to publish it and make money from it.


You were right; it's someone else who is selling Siklos' book. He wrote a review.
Original post by DomStaff
What was the other year?


This question was quite tricky

Arthur and Bertha stand at a point O on an inclined plane. The steepest line in the plane through O makes an angle θ\theta with the horizpontal. Arthur walks uphill at a steady pace in a straight line which makes an angle α\alpha with the steepest line. Bertha walks uphill at the same speed in a straight line which makes an angle β\beta with the steepest line (and is on the same side of the steepest line as Arthur) Show that, when Arthur has walked a distancde dd, the distance between Arthur and Bertha is 2dsin12(αβ)2d|\sin\frac{1}{2}(\alpha-\beta)|. Show also that, if αβ\alpha\not=\beta, the line joining arthur and Bertha makes an angle ϕ with the vertical, where cosϕ=sinθsin12(α+β)\phi\text{ with the vertical, where }\cos\phi=\sin\theta\sin\frac{1}{2}(\alpha+\beta)
Can someone explain to me the last part of Question 12, Advanced Problems in Mathematics Siklos Booklet. It's the questions that asks:

Show by means of counterexample or otherwise, that not all cubic equations of the form

x3+αx2+βx+γ=0x^{3} + \alpha x^{2} + \beta x^{} + \gamma = 0

can be solved by this method
Reply 392
Original post by Godel_Mark
Can someone explain to me the last part of Question 12, Advanced Problems in Mathematics Siklos Booklet. It's the questions that asks:

Show by means of counterexample or otherwise, that not all cubic equations of the form

x3+αx2+βx+γ=0x^{3} + \alpha x^{2} + \beta x^{} + \gamma = 0

can be solved by this method


The question is a bit weird, the discussion below does give an example but I'm not how you are expected think of it on your own. Granted the question itself is pretty old (II 1989), so the style is a bit different from the more recent questions - I don't think you'll be asked something like this anymore.

Although it's a lot of work and it's probably not ideal if this question comes up in the actual exam, you can try the method on an arbitrary cubic and see what the requirements are to get it in that form. It's actually pretty interesting, you can express the valid coefficients in the form of an inequality.

Another possibility that just occurred to me is to consider what happens if there are complex roots, looking back at the example cubic you just solved. Still not exactly easy to come up with, but hopefully that also gives you an idea of why the example they listed doesn't work.
Original post by ThatPerson
You were right; it's someone else who is selling Siklos' book. He wrote a review.


Yes i sent him an e mail alerting him about this earlier on today.


Posted from TSR Mobile
Original post by brianeverit
Yes. when you have 1+x^2 to get rid of.
BY the way, what were your two answers?
This was an interesting one.
Two points P and Q lie within, or on the boundary of, a square of side 1 cm, one corner of which is the point O. Show that the length of at least one of the lines OP,PQ and QO must be less than or equal to (62)(\sqrt6-\sqrt2) cm


You suggested the same one as me :smile: cool
Original post by physicsmaths
Yes i sent him an e mail alerting him about this earlier on today.


Posted from TSR Mobile


Lol it sounds like someones trying to get on his good side :colone:

I wonder what college you're applying to:rolleyes:
https://share.trin.cam.ac.uk/sites/public/Tutorial/Admissions-UG/sampletest1.pdf

Q9) I think it's unlikely this is what they were looking for but it seems reasonable:

1+ln(n+1)>Hn>ln(n) 1 + \ln(n + 1) > H_n > \ln(n) so approximate y=ln(w) y = \ln(w) by
Unparseable latex formula:

H_[_w_]

, [w][ln(M)] [w] \approx [ \ln(M) ] so
Unparseable latex formula:

y \approx H_[_\ln(M)_]

where [x] [x] is the nearest integer to x x

Does anyone have another solution?
Original post by jjpneed1
Does anyone have another solution?


A natural manipulation gives w=lnM+ln(lnM+ln(lnM+))w=\ln M+\ln(\ln M+\ln(\ln M+\cdots)\cdots ) i.e. y=ln(lnM+ln(lnM+))y=\ln(\ln M+\ln(\ln M+\cdots)\cdots ) [this nested expression converges to the exact value]
Reply 398
Original post by physicsmaths
Yes i sent him an e mail alerting him about this earlier on today.


I shot Amazon an email as well. Interesting to see if they'll do anything about it...
Original post by newblood
Lol it sounds like someones trying to get on his good side :colone:

I wonder what college you're applying to:rolleyes:


hahaha lol. Just trying to be nice
............... 😜


Posted from TSR Mobile

Quick Reply

Latest

Trending

Trending